Vector divide with negative ratio?

Click For Summary
The discussion revolves around finding the position vector of point P that divides the line segment AB in the ratio 5:-2, with points A (-1,6,4) and B (4,1,-1). There is confusion regarding the interpretation of the negative ratio, with participants questioning whether it implies P is outside the segment AB or if it indicates a specific direction. The terminology used in vector ratios is noted as unusual, prompting further clarification on its meaning. The participants are trying to reconcile the standard interpretation of ratios with the vector context presented in their homework. The conversation highlights the complexities of applying the Ratio Theorem in vector mathematics.
terryds
Messages
392
Reaction score
13

Homework Statement


[/B]
Find the position vector of point P if P divides AB in the ratio 5:-2 given A (-1,6,4) and B (4,1,-1)

The Attempt at a Solution



The only trouble I have is to understand the question...

Does it mean something like

A------------------>B<----------------------P
3 2

Or

A----------------->P<-----------------------B
5 2Please help
 
Physics news on Phys.org
What strange terminology! I've not seen anybody use ratios with vectors that way before.
I can only guess that when they say 'P divides AB in the ratio r:s' they mean
$$\frac{\vec{AP}}{r}=\frac{\vec{PB}}{s}$$

Which of your two options does that favour?
 
andrewkirk said:
What strange terminology! I've not seen anybody use ratios with vectors that way before.
I can only guess that when they say 'P divides AB in the ratio r:s' they mean
$$\frac{\vec{AP}}{r}=\frac{\vec{PB}}{s}$$

Which of your two options does that favour?

Hmm... I just don't understand what the negative sign means...
Is it really strange to use ratio with vectors? It's on the subchapter "Ratio Theorem" in Vector chapter.
 
Question: A clock's minute hand has length 4 and its hour hand has length 3. What is the distance between the tips at the moment when it is increasing most rapidly?(Putnam Exam Question) Answer: Making assumption that both the hands moves at constant angular velocities, the answer is ## \sqrt{7} .## But don't you think this assumption is somewhat doubtful and wrong?

Similar threads

Replies
2
Views
8K
Replies
9
Views
2K
  • · Replies 2 ·
Replies
2
Views
2K
  • · Replies 1 ·
Replies
1
Views
2K
Replies
1
Views
1K
  • · Replies 24 ·
Replies
24
Views
3K
  • · Replies 16 ·
Replies
16
Views
2K
  • · Replies 2 ·
Replies
2
Views
1K
Replies
1
Views
1K
Replies
11
Views
4K